1966 AHSME Problems/Problem 15

Revision as of 22:11, 14 January 2018 by Bobsonjoe (talk | contribs) (Solution)
(diff) ← Older revision | Latest revision (diff) | Newer revision → (diff)

Problem

If $x-y>x$ and $x+y<y$, then

$\text{(A) } y<x \quad \text{(B) } x<y \quad \text{(C) } x<y<0 \quad \text{(D) } x<0,y<0 \quad \text{(E) } x<0,y>0$

Solution

From $x-y>x$, we get that $-y>0\implies y<0$. From $x+y<y$, we get that $x<0$. So, our final answer is $\fbox{D}$.

See also

1966 AHSME (ProblemsAnswer KeyResources)
Preceded by
Problem 14
Followed by
Problem 16
1 2 3 4 5 6 7 8 9 10 11 12 13 14 15 16 17 18 19 20 21 22 23 24 25 26 27 28 29 30
All AHSME Problems and Solutions

The problems on this page are copyrighted by the Mathematical Association of America's American Mathematics Competitions. AMC logo.png